LSAT and Law School Admissions Forum

Get expert LSAT preparation and law school admissions advice from PowerScore Test Preparation.

User avatar
 Dave Killoran
PowerScore Staff
  • PowerScore Staff
  • Posts: 5852
  • Joined: Mar 25, 2011
|
#55799
Complete Question Explanation
(The complete setup for this game can be found here: lsat/viewtopic.php?t=23743)

The correct answer choice is (E)

This question is identical to question #1, with the exception that W has been dropped from the ordering (but, of course, the rules involving W are still active). Thus, you should approach this question in the same fashion as any List question.

Answer choice (A): This answer specifies that Patterson meets with Y before S, but, as we know from the combination of the first and second rule, Patterson cannot meet with Y before S, and thus this answer choice is incorrect.

Answer choice (B): This answer specifies that Patterson meets with T before Y, but, as we know from the combination of the second and third rule, Patterson cannot meet with T before Y, and thus this answer choice is incorrect.

Answer choice (C): This answer violates the fourth rule because the meeting with U is not ahead of the meeting with R.

Answer choice (D): Like answer choice (B), this answer specifies that Patterson meets with T before Y, but since Patterson cannot meet with T before Y, this answer choice is incorrect.

Answer choice (E): This is the correct answer choice.

Get the most out of your LSAT Prep Plus subscription.

Analyze and track your performance with our Testing and Analytics Package.